Đến nội dung

hoanglong2k nội dung

Có 992 mục bởi hoanglong2k (Tìm giới hạn từ 30-03-2020)



Sắp theo                Sắp xếp  

#645323 ĐỀ THI OLYMPIC GẶP GỠ TOÁN HỌC LẦN VIII

Đã gửi bởi hoanglong2k on 17-07-2016 - 19:47 trong Thi HSG cấp Tỉnh, Thành phố. Olympic 30-4. Đề thi và kiểm tra đội tuyển các cấp.

Bài 2.

Ta đặt $x=a+b$ và $y=a+c$ thì $3a+2b+c=2x+y$ với $x,y\geq 0$

 Chú ý là ta có các đánh giá $a+b+c\leq 2a+b+c=x+y$ và $ab+bc+ca\leq a^2+ab+bc+ca=(a+b)(a+c)=xy$, cho nên ta chỉ cần chứng minh

\[(2x+y)^3\geq 6\sqrt{3}xy(x+y)\]

 Chuẩn hóa $y=1$ thì ta cần chứng minh $8x^3+(12-6\sqrt{3})x^2+(6-6\sqrt{3})x+1\geq 0$

 Xét hàm số $f(x)=8x^3+(12-6\sqrt{3})x^2+(6-6\sqrt{3})x+1$ trên $[0;\infty )$

 Có $f'(x)=24x^2+12(2-\sqrt{3})x+6-6\sqrt{3}$, phương trình $f'(x)=0\Leftrightarrow 6(2x+1)(2x+1-\sqrt{3})=0\Leftrightarrow x=\dfrac{\sqrt{3}-1}{2}$

 Từ đó ta có thể suy ra được $f(x)\geq f\left(\dfrac{\sqrt{3}-1}{2}\right)=0$

 Bài toán được chứng minh. Dấu "=" xảy ra khi $a=0$ và $b=\dfrac{\sqrt{3}-1}{2}c$




#639989 VMF's Marathon Bất Đẳng Thức Olympic

Đã gửi bởi hoanglong2k on 13-06-2016 - 08:40 trong Bất đẳng thức và cực trị

Bài toán 32.  (Vasile Cirtoaje) Cho các số thực dương $a,b,c,d$. Chứng minh rằng 

$$2(a^3+1)(b^3+1)(c^3+1)(d^3+1) \geqslant (1+abcd)(1+a^2)(1+b^2)(1+c^2)(1+d^2)$$

 Lời giải bài 32. Cho $a=b=c=d$ thì ta cần chứng minh $2(a^3+1)^4\geq (1+a^4)(1+a^2)^4$ với mọi $a\in \mathbb{R^+}$

 Biến đổi thành $(a-1)^2(a^{10}+2a^9-a^8+4a^7+2a^6+2a^4+4a^3-a^2+2a+1)\geq 0$

 Mà $a^{10}+2a^9-a^8+4a^7+2a^6+2a^4+4a^3-a^2+2a+1$

       $=a^{10}+a^7(2a^2-a+4)+2a^6+2a^4+a(4a^2-a+2)+1>0$

 Cho nên $2(a^3+1)^4\geq (1+a^4)(1+a^2)^4$, tương tự thì $2(b^3+1)^4\geq (1+b^4)(1+b^2)^4,\cdots $

 Nhân lại ta thu được $2^4(a^3+1)^4(b^3+1)^4(c^3+1)^4(d^3+1)^4$

                                  $\geq (1+a^4)(1+b^4)(1+c^4)(1+d^4)(1+a^2)^4(1+b^2)^4(1+c^2)^4(1+d^2)^4$ 

 Áp dụng bất đẳng thức Holder ta có $(1+a^4)(1+b^4)(1+c^4)(1+d^4)\geq (1+abcd)^4$

 Từ đó ta suy ra điều cần chứng minh

 Tương tự ta có thể mở rộng lên cho $n$ số cũng được :)

 

 Bài toán 33. (AoPS) Cho các số thực $a,b,c$ thỏa mãn $a,b,c\geq 1$ và $a+b+c+2=abc$. Chứng minh rằng

\[\dfrac{\sqrt{ab-1}+\sqrt{bc-1}+\sqrt{ca-1}}{\sqrt[4]{abc}}\leq \dfrac{1}{2}(\sqrt{a}+\sqrt{b}+\sqrt{c})\sqrt{\dfrac{1}{\sqrt{a}}+\dfrac{1}{\sqrt{b}}+\dfrac{1}{\sqrt{c}}}\]




#639825 $\sum \sqrt{k^2a^2+bc}\leq \dfrac{(2k...

Đã gửi bởi hoanglong2k on 12-06-2016 - 14:21 trong Bất đẳng thức - Cực trị

Bất đẳng thức này đúng với mọi số thực $k \geqslant 1.$

 Anh có thể cho lời giải tổng quát được không ạ :) Em mới chỉ chứng minh được cho $k$ từ $1$ đến xấp xỉ $3,0403022...$




#639555 $\sum \sqrt{k^2a^2+bc}\leq \dfrac{(2k...

Đã gửi bởi hoanglong2k on 11-06-2016 - 11:17 trong Bất đẳng thức - Cực trị

Bài toán. Cho $a,b,c$ là các số thực không âm, chứng minh rằng với mọi số thực $k\in \big [1,3\big ]$ ta luôn có bất đẳng thức sau

\[\sqrt{k^2a^2+bc}+\sqrt{k^2b^2+ca}+\sqrt{k^2c^2+ab}\leq \dfrac{(2k+1)}{2}(a+b+c)\]

 

 

...




#639547 VMF's Marathon Bất Đẳng Thức Olympic

Đã gửi bởi hoanglong2k on 11-06-2016 - 10:38 trong Bất đẳng thức và cực trị

 Bài toán 29. (Võ Quốc Bá Cẩn) Cho $a,b,c$ là các số thực không âm chứng minh

$\sqrt{4a^{2}+bc}+\sqrt{4b^{2}+ca}+\sqrt{4c^{2}+ab}\leq \frac{5}{2}(a+b+c)$

 Ý tưởng giải bài này của mình dựa trên một kết quả tương tự của Phạm Kim Hùng, sau đây là lời giải của mình cho bài 29

 Lời giải bài 29. Không mất tính tổng quát, giả sử $a\geq b\geq c$.

 Áp dụng bất đẳng thức Cauchy-Schwarz ta có $\sqrt{4b^2+ca}+\sqrt{4c^2+ab}\leq \sqrt{(2+1)\left(\dfrac{4b^2+ca}{2}+4c^2+ab\right)}=\sqrt{\dfrac{3}{2}\left(4b^2+ca+8c^2+2ab\right)}$

 Lại có $\sqrt{4a^2+bc}\leq \sqrt{4a^2+ac}\leq 2a+\dfrac{c}{4}$

 Nên ta chỉ cần chứng minh $\sqrt{\dfrac{3}{2}\left(4b^2+ca+8c^2+2ab\right)}\leq \dfrac{a}{2}+\dfrac{5b}{2}+\dfrac{9c}{4}$

 Hay là $\sqrt{24\left(4b^2+ac+8c^2+2ab\right)}\leq 2a+10b+9c$

 Bình phương 2 vế thì ta cần chứng minh $(2a+10b+9c)^2\geq 24(4b^2+ac+8c^2+2ab)\Leftrightarrow (2a-2b+3c)^2+24c(8b-5c)\geq  0$

 Bất đẳng thức trên đúng do $a\geq b\geq c$ nên ta có điều cần chứng minh

 Dấu "=" xảy ra khi $a=b,c=0$ hoặc các hoán vị

 

 Bài 28 không biết có thể giải bằng dồn biến không nhỉ, mình có thử nhưng mới chỉ chứng minh được một TH nhỏ :(




#638611 Cập nhật tình hình, thảo luận, chém gió về kì thi vào lớp 10 THPT

Đã gửi bởi hoanglong2k on 06-06-2016 - 22:21 trong Góc giao lưu

Kinh nghiệm học văn ôn thi vào cấp 3 theo mình là học hết còn hơn bỏ xót :v năm ngoái gần 2 tháng cày văn :v đi thi mong văn trên điểm liệt cuối cùng điểm văn được 8 :v bất ngờ :))

 

nghe thấy tự hào mình ghê :v hồi đấy được 8,75 :3

 Hơi nhột rồi đấy nhé -_-




#638606 VMF's Marathon Bất Đẳng Thức Olympic

Đã gửi bởi hoanglong2k on 06-06-2016 - 22:07 trong Bất đẳng thức và cực trị

Một lời giải bài toán 17: Thực chất bài tổng quát của bài toán 17 bài toán sau

$x^{n}y+y^{n}z+z^{n}x\leq \frac{n^{n}}{(n+1)^{n+1}}(x+y+z)^{n+1}$$\forall x,y,z\geq 0$

Ta chỉ cần chứng minh bài toán tổng quát .

Với $n=1$, bất đẳng thức đúng.

Với $n> 1$: Không mất tính tổng quát, giả sử $x=\max \left \{ x,y,z \right \}$ . Ta có

$\left\{\begin{matrix} y\leq x\Rightarrow y^{n}z\leq x^{n-1}yz & & \\ z\leq x\Rightarrow z^{n}x\leq zx^{n} & & \\ z^{n}x\leq z^{2}x^{n-1}& & \\ n> 1\Rightarrow \frac{n-1}{n}\geq \frac{1}{2}\Rightarrow \frac{n-1}{n}z\geq \frac{z}{2}& & \end{matrix}\right.$

Đặt $P=x^{n}y+y^{n}z+z^{n}x\leq x^{n}y+x^{n-1}yz+\frac{1}{2}z^{n}x+\frac{1}{2}z^{n}x$

    $\leq x^{n}y+x^{n-1}yz+\frac{1}{2}x^{n}z+\frac{1}{2}z^{2}x^{n-1}$

    $=x^{n-1}(x+z)\left ( y+\frac{z}{2} \right )$

    $\leq x^{n-1}(x+z)(y+\frac{n-1}{n}z)=n^{n}[\frac{x}{n}.\frac{x}{n}...\frac{x}{n}.\frac{x+z}{n}(y+\frac{n-1}{n}z)]$

    $\leq$$n^{n}[\frac{(n-1).\frac{x}{n}+\frac{x+z}{n}+y+\frac{n-1}{n}z}{n=1}]^{n+1}$

    $=\frac{n^{n}}{(n+1)^{n+1}}(x+y+z)^{n+1}$

Đẳng thức xảy ra khi và chỉ khi $x=y=z$ hoặc $x=y,z=0$ và các hoặc các hoán vị của nó

Áp dụng ta có được $x^{3}y+y^{3}z+z^{3}x\leq \frac{27}{256}(x+y+z)^{4}$

 Cảm ơn bạn, đây thực chất chỉ là một kết quả cũ :) Xin trình bày cách của mình

 Không mất tính tổng quát, giả sử $y$ nằm giữa $x$ và $z$ thì ta có $z\left(y^{n-1}-z^{n-1}\right)(y-x)\leq 0\Leftrightarrow y^nz+xz^n\leq yz^n+xy^{n-1}z$

 Do đó ta có được $x^ny+y^nz+z^nx\leq y\left(x^n+z^n+xy^{n-2}z\right)$

 Chú ý là từ khai triển nhị thức $(x+z)^n=x^n+z^n+nxz\left(x^{n-2}+z^{n-2}\right)+...\geq x^n+z^n+nxz\left(x^{n-2}+z^{n-2}\right)\geq x^n+z^n+nxzy^{n-2}$

 Cho nên

$x^ny+y^nz+z^nx\leq y(x+z)^n=n^n.y.\dfrac{x+z}{n}.\dfrac{x+z}{n}\cdots \dfrac{x+z}{n}$

$\leq n^n\left(\dfrac{y+x+z}{n+1}\right)^{n+1}=\dfrac{n^n}{(n+1)^{n+1}}(x+y+z)^{n+1}$

 Vả lại dấu "=" của bạn nhầm rồi, ở đây là khi $(x,y,z)\sim (n,1,0)$ cùng các hoán vị tương ứng




#638552 Cập nhật tình hình, thảo luận, chém gió về kì thi vào lớp 10 THPT

Đã gửi bởi hoanglong2k on 06-06-2016 - 18:39 trong Góc giao lưu

mà nghĩ cũng lạ, học toán giỏi thì dốt văn, ai trả lời giùm em câu này.(mai thi rồi mà chả có một chữ văn , cầu mong ra thơ việt nam) 

 Ai bảo thế :) Xa xưa thì có Trạng Lường Lương Thế Vinh, rồi Lê Quý Đôn, nay thì VMF ta có thầy Thế, rồi thầy Thanh, học sinh có anh kudoshinichihv99 gì đó nữa mà nhỉ 




#638506 VMF's Marathon Bất Đẳng Thức Olympic

Đã gửi bởi hoanglong2k on 06-06-2016 - 14:53 trong Bất đẳng thức và cực trị

Nguyenngoctu

 

fatcat12345

  Tiện thể mình sẽ đề xuất bài toán 27 để chúng ta tiếp tục Topic :)

 Bài toán 27. (Sưu tầm) Chứng minh rằng với các số thực $x\geq 0$ tùy ý và số nguyên dương $n$ ta luôn có bất đẳng thức \[\left[nx\right]\geq \dfrac{\left[x\right]}{1}+\dfrac{\left[2x\right]}{2}+\dfrac{\left[3x\right]}{3}+\cdots +\dfrac{\left[nx\right]}{n}\]




#638414 Cập nhật tình hình, thảo luận, chém gió về kì thi vào lớp 10 THPT

Đã gửi bởi hoanglong2k on 05-06-2016 - 22:50 trong Góc giao lưu

Mà nhớ lại hồi đấy thì có lẽ trong những đứa đậu vào Toán 1 của Chuyên Quốc Học Huế thì mình là thằng có điểm văn thấp nhất =)) (không chỉ của QB mà của cả những đứa đậu luôn chứ), ngẫm cũng lạ, chế thơ hay thế mà mấy ông chấm gắt quá :))




#638413 Cập nhật tình hình, thảo luận, chém gió về kì thi vào lớp 10 THPT

Đã gửi bởi hoanglong2k on 05-06-2016 - 22:48 trong Góc giao lưu

Mỗi Văn ngu thì có gì phải sợ, hồi lớp 9 anh đây Văn Anh max ngu, điểm Văn $3,75$ còn Anh $4,8$ thế mà vẫn đậu lớp Toán 1 :D

Nhớ lại thì hồi đó thi Văn mà không chịu học thuộc thơ, thế là cả bài văn $4,5$ điểm gì đấy ngậm bút không biết viết cái gì, mà Văn mình vào đó cũng không bằng một nửa họ, chúng nó chém $3$ tờ giấy trong $90$ phút, mình "rặn" mãi $3$ mặt giấy mà hết $60$ phút rồi, $30$ phút còn lại ngồi ngắm trường :D

 Quá ngon rồi, văn 3,5 điểm đây :D Toán 1 Tuấn nhỉ :-|




#637369 VMF's Marathon Bất Đẳng Thức Olympic

Đã gửi bởi hoanglong2k on 01-06-2016 - 12:43 trong Bất đẳng thức và cực trị

 

Bài toán 22. (AoPS) Cho các số thực dương $a,b,c,d$. Chứng minh rằng
\[ a^4+b^4+c^4+d^4-4abcd\geq 2a^2|(d-b)(d-c)|.\]

 

 Lời giải bài 22.

 - Trường hợp $(d-b)(d-c)\leq 0$,ta viết bất đẳng thức lại là $a^4+b^4+c^4+d^4-4abcd\geq 2a^2(d-b)(c-d)$

   Áp dụng bất đẳng thức AM-GM ta có $(d-b)(c-d)\leq \dfrac{(b-c)^2}{4}$

   Nên ta chỉ cần chứng minh $a^4+b^4+c^4+d^4-4abcd\geq \dfrac{a^2(b-c)^2}{2}$

   Tương đương $2(a^4+b^4+c^4+d^4)+2a^2bc\geq a^2b^2+a^2c^2+8abcd$

   Áp dụng bất đẳng thức AM-GM ta có $a^2bc+a^2bc+d^4+b^2c^2\geq 4abcd$ và $a^4+b^4+c^4+d^4\geq 4abcd$

   Nên ta chỉ cần chứng minh $a^4+b^4+c^4\geq a^2b^2+b^2c^2+c^2a^2$, hiển nhiên đúng

 - Trường hợp $(d-b)(d-c)\geq 0$, áp dụng bất đẳng thức AM-GM ta sẽ có $b^4+c^4\geq \dfrac{(b+c)^4}{8},bc\leq \dfrac{(b+c)^2}{4},(d-b)(d-c)\leq \dfrac{(2d-b-c)^2}{4}$

   Đặt $t=\dfrac{b+c}{2}$ thì ta chỉ cần chứng minh $a^4+2t^4+d^4\geq 2a^2(d-t)^2$

   Tương đương với $(a^2-d^2)^2+2t(t-a)(t^2+at-2ad)\geq 0$

   Hiển nhiên với $t=\max \{a,t,d\}$ hoặc $t=\min \{a,t,d\}$  thì bất đẳng thức trên đúng nên ta chỉ xét khi $t$ nằm giữa $a$ và $d$

   Nếu $a\geq t\geq d$, đặt $a=d+x$ và $t=d+y$ thì ta cần chứng minh $(x^2+2dx)^2+2(d+y)(y-x)(3dy+y^2+xy-dx)\geq 0$

   Tương đương với $2d^2(3x^2-4xy+3y^2)+4d(x^3-2xy^2+2y^3)+x^4-2x^2y^2+2y^4\geq 0$

   Hiển nhiên đúng do $3x^2-4xy+3y^2\geq x^2+y^2\geq 0,x^3-2xy^2+2y^3\geq xy^2\geq 0,x^4-2x^2y^2+2y^4\geq y^4\geq 0$

   Nếu $a\leq t\leq d$, đặt $d=a+u,t=a+v$ thì ta cần chứng minh $a^2(4u^2-4uv+6v^2)+4a(u^3-uv^2+2v^3)+u^4+2v^4\geq 0$, đúng

 Vậy ta có điều cần chứng minh. Dấu "=" xảy ra khi $a=b=c=d$

 

 Bài toán 23. (Sưu tầm) Cho các số thực không âm $x,y,z$ thỏa mãn $x^2+y^2+z^2+2xyz=1$. Chứng minh rằng

\[x^3y+y^3z+z^3x\leq \dfrac{3\sqrt{3}}{16}\]




#636983 VMF's Marathon Bất Đẳng Thức Olympic

Đã gửi bởi hoanglong2k on 31-05-2016 - 00:09 trong Bất đẳng thức và cực trị

Trong lời giải này, bước chuyển pqr cần tính toán khá nhiều, chứ không phải một bước ra luôn như dùng phần mềm máy tính. Mình nghĩ trong topic này chỉ nên đăng các bất đẳng thức và lời giải có thể tính toán bằng tay thôi.
 

Xin được đề xuất bài toán sau:

Bài toán 20. (AoPS)  Cho các số thực không âm $a,b,c$ thỏa mãn  $a+b+c=3.$ Chứng minh rằng 

\[ a^3b^2+b^3c^2+c^3a^2+7(ab+bc+ca) \leq 24.\]

 Một lời giải "xấu xí" của em cho bài toán này

 Lời giải bài 20.

 Tương tự như anh Quý, viết lại bất đẳng thức về dạng đồng bậc là 

$$81(a^3b^2+b^3c^2+c^3a^2)+21(ab+bc+ca)(a+b+c)^3\leq 8(a+b+c)^5$$

 Không mất tính tổng quát, giả sử $b$ nằm giữa $a$ và $c$, khi đó ta sẽ có $c(bc-c^2)(b^2-a^2)=c^2(b+a)(b-c)(b-a)\leq 0$

 Do đó mà $b^3c^2+c^3a^2\leq b^2c^3+a^2bc^2$

 Ta cần chứng minh $81b\left[b(a^3+c^3)+a^2c^2\right]+21\left[b(c+a)+ca\right](a+b+c)^3\leq 8(a+b+c)^5$

 Tương đương $81b\left[b(a+c)^3-3abc(a+c)+a^2c^2\right]+21\left[b(c+a)+ca\right](a+b+c)^3\leq 8(a+b+c)^5$

 Chuẩn hóa $a+b+c=1$ và đặt $t=ac$ thì do $(b-a)(b-c)\leq 0\Rightarrow t\leq b(a+c-b)=b(1-2b)$, từ đây cũng suy ra $b\leq \dfrac{1}{2}$

 Ta cần chứng minh

$$81b\left[b(1-b)^3-3bt(1-b)+t^2\right]+21\left[b(1-b)+t\right]\leq 8$$

$$\Leftrightarrow 81b(-b^4+3b^3-3b^2+b-3bt+3b^2t+t^2)+21(-b^2+b+t)\leq 8$$

$$\Leftrightarrow 81bt^2+t(243b^3-243b^2+21)\leq 81b^5-243b^4+243b^3-60b^2-21b+8$$

 Mà $t\leq b(1-2b)$ nên ta chỉ cần chứng minh

$$81tb(1-2b)+t(243b^3-243b^2+21)\leq 81b^5-243b^4+243b^3-60b^2-21b+8$$

$$\Leftrightarrow 3t(3b+1)(9b^2-21b+7)\leq 81b^5-243b^4+243b^3-60b^2-21b+8$$

 Bây giờ, nếu $9b^2-21b+7\leq 0$ tức là $b\geq \dfrac{7-\sqrt{21}}{6}>0,4$

 Khi này ta chỉ cần chứng minh $81b^5-243b^4+243b^3-60b^2-21b+8\geq 0$ với $b\in \left[\dfrac{7-\sqrt{21}}{6};\dfrac{1}{2}\right]$ là đủ (thực ra nó đúng với mọi $b\geq 0$ thì phải)

 Đoạn này mình chứng minh không được đẹp lắm, phải chia nhỏ ra để làm

 Đặt $f(b)=81b^5-243b^4+243b^3-60b^2-21b+8$ với $b\in \left[\dfrac{7-\sqrt{21}}{6};\dfrac{1}{2}\right]$

 Có $f'(b)=3(135b^4-324b^3+243b^2-40b-7),\ f''(b)=6(270b^3-486b^2+243b-20)>0$ nên $f'(b)$ đồng biến.

  Trường hợp $0,4<b\leq 0,443$ thì $f'(b)\leq f'(0,443)<0$ nên $f(b)\geq f(0,443)>0$

  Trường hợp $0,5\geq b\geq 0,4431$ thì $f'(b)\geq f'(0,4431)>0$ nên $f(b)\geq f(0,4431)>0$

  Còn mà $0,443<b<0,4431$ thì $81b^5-243b^4+243b^3-60b^2-21b+8$

                                                      $\geq 81.0,443^5-243.0,4431^4+243.0,443^3-60.0,4431^2-21.0,4431+8>0$

 Do đó $f(b)>0$ và trường hợp này đúng

 Nếu $9b^2-21b+7\geq 0$, do $t\leq b(1-2b)$ nên ta chỉ cần chứng minh

$$3b(1-2b)(3b+1)(9b^2-21b+7)\leq 81b^5-243b^4+243b^3-60b^2-21b+8$$

$$\Leftrightarrow (3b-1)^2(27b^3-54b^2+6b+8)\geq 0$$

 Bất đẳng thức trên đúng với mọi $0\leq b\leq \dfrac{1}{2}$ nên ta có điều cần chứng minh

 Dấu "=" xảy ra khi và chỉ khi $a=b=c=1$




#636674 VMF's Marathon Bất Đẳng Thức Olympic

Đã gửi bởi hoanglong2k on 29-05-2016 - 22:30 trong Bất đẳng thức và cực trị

Lời giải bài 18. Ta viết bất đẳng thức thành $\dfrac{\sum a^4+3\sum a^2b^2}{\prod (a^2+b^2)}\geq \dfrac{4}{a^2+b^2+c^2}+\dfrac{1}{2(ab+bc+ca)}$

 Đặt $p=a+b+c=1,q=ab+bc+ca,r=abc$ thì ta cần chứng minh $\dfrac{1-4q+5q^2-2r}{q^2-2q^3-2r+4qr-r^2}\geq \dfrac{1+6q}{2q(1-2q)}$

 Hay là $r^2(1+6q)+r(1+4q)(1-2q)+q(2-q)(1-4q)(1-2q)\geq 0$

 Nếu $q\leq \dfrac{1}{4}$ thì $r^2(1+6q)+r(1+4q)(1-2q)+q(2-q)(1-4q)(1-2q)\geq q(2-q)(1-4q)(1-2q)\geq 0$

 Nếu $\dfrac{1}{3}\geq q\geq \dfrac{1}{4}$, áp dụng bất đẳng thức Schur bậc 4 ta có $r\geq \dfrac{(4q-1)(1-q)}{6}$

 Do đó ta chỉ cần chứng minh $\left[\dfrac{(4q-1)(1-q)}{6}\right]^2(1+6q)+\dfrac{(4q-1)(1-q)(1+4q)(1-2q)}{6}+q(2-q)(1-4q)(1-2q)\geq 0$

 Tương đương với $(3q-1)(4q-1)(8q^3-6q^2+27q-11)\geq 0$, tức là ta chỉ cần chứng minh $f(q)=8q^3-6q^2+27q-11\leq 0$ với $\dfrac{1}{3}\geq q\geq \dfrac{1}{4}$

 Mặt khác, $f'(q)=24q^2-12q+27>0$ nên $f(q)\leq f\left(\dfrac{1}{3}\right)=\dfrac{-64}{27}<0$

 Do đó ta có điều cần chứng minh. Dấu "=" xảy ra khi $a=b=c$ hoặc $a=b,c=0$ cùng các hoán vị

 

 Một lời giải khác cho bài 17.

 Một lời giải khác nữa :)

 

  Bài toán 19. (AoPS) Cho $x,y,z\geq 0$ thỏa mãn $x+y+z=32$. Chứng minh rằng

\[x^3y+y^3z+z^3x+\dfrac{473xyz}{8}\leq 110592\]

 

 \begin{array}{| l | l |} \hline \text{HDTterence2k} & 1\\ \hline \text{hoanglong2k} & 5\\ \hline \text{Gachdptrai12} & 8\\ \hline \text{Nguyenhuyen_AG} & 6\\ \hline \text{fatcat12345} & 5\\ \hline \text{lenhatsinh3} & 1\\ \hline \text{tuanyeubeo2000} & 1\\ \hline \text{Ngockhanh99k48} & 1 \\ \hline \text{Dinh de Tai} & 1\\ \hline \end{array} 




#636580 VMF's Marathon Bất Đẳng Thức Olympic

Đã gửi bởi hoanglong2k on 29-05-2016 - 17:23 trong Bất đẳng thức và cực trị

 Lời giải bài 16. Giả sử $c=\min \{a,b,c\}$. Trước tiên ta chứng minh $\dfrac{1}{\sqrt{b^2+bc+c^2}}+\dfrac{1}{\sqrt{a^2+ac+c^2}}\geq \dfrac{2\sqrt{2}}{\sqrt{2ab+ac+bc+2c^2}}$

Tương đương với 

$\begin{align*}2\left(\dfrac{1}{b^2+bc+c^2}+\dfrac{1}{a^2+ac+c^2}-\dfrac{4}{2ab+ac+bc+2c^2}\right)&\geq \left(\dfrac{1}{\sqrt{b^2+bc+c^2}}-\dfrac{1}{\sqrt{a^2+ac+c^2}}\right)^2\\ \Leftrightarrow \dfrac{2(a-b)^2(2ab+bc+ca-c^2)}{(a^2+ac+c^2)(b^2+bc+c^2)(2ab+ac+bc+2c^2)}&\geq \dfrac{(a-b)^2(a+b+c)^2}{(a^2+ac+c^2)(b^2+bc+c^2)\left(\sqrt{a^2+ac+c^2}+\sqrt{b^2+bc+c^2}\right)^2}\\ \Leftrightarrow \dfrac{2(2ab+bc+ca-c^2)}{2ab+ac+bc+2c^2}&\geq \dfrac{(a+b+c)^2}{\left(\sqrt{a^2+ac+c^2}+\sqrt{b^2+bc+c^2}\right)^2} \end{align*}$

 Hiển nhiên đúng do $\dfrac{2(2ab+bc+ca-c^2)}{2ab+ac+bc+2c^2}\geq 1\geq \dfrac{(a+b+c)^2}{\left(\sqrt{a^2+ac+c^2}+\sqrt{b^2+bc+c^2}\right)^2}$

 Do đó $\text{VT}\geq \dfrac{2\sqrt{2}}{\sqrt{(a^2+ab+b^2)(2ab+ac+bc+2c^2)}}+\dfrac{1}{\sqrt{(c^2+ca+a^2)(c^2+cb+b^2)}}$

 Đặt $t=ab$ và xét $f(t)=\dfrac{2\sqrt{2}}{\sqrt{\left[(1-c)^2-t\right](2t+c^2+c)}}$

 Có $f'(t)=\dfrac{\sqrt{2}(4t+5c-c^2-2)}{(c^2-2c-t+1)(c^2+c+2t)\sqrt{(c^2-2c-t+1)(c^2+c+2t)}}$

 Mà $4t+5c-c^2-2\leq (a+b)^2+5c-c^2-2=(1-c)^2+5c-c^2-2=3c-1\leq 0$ nên $f(t)$ nghịch biến

 Suy ra $f(t)\geq f\left [\dfrac{(a+b)^2}{4}\right]=\dfrac{8}{\sqrt{3(a+b)^2\left[4c^2+2c(a+b)+(a+b)^2\right]}}=\dfrac{8}{\sqrt{3(1-c)^2(1+3c^2)}}$

 Mặt khác lại có $\left[4c^2+2c(a+b)+(a+b)^2\right]^2-16(a^2+ac+c^2)(b^2+bc+c^2)$

                          $=(a-b)^2(a^2+6ab+4ac+b^2+4bc-4c^2)\geq 0$

 Cho nên $\dfrac{1}{\sqrt{(c^2+ca+a^2)(c^2+cb+b^2)}}\geq \dfrac{4}{4c^2+2c(a+b)+(a+b)^2}=\dfrac{4}{3c^2+1}$

 Vậy nên $\text{VT}\geq \dfrac{8}{\sqrt{3(1-c)^2(1+3c^2)}}+\dfrac{4}{3c^2+1}$

 Xét $g(c)=\dfrac{8}{\sqrt{3(1-c)^2(1+3c^2)}}+\dfrac{4}{3c^2+1}$ trên $\left[0,\dfrac{1}{3}\right]$ ta được $\min g(c)=g(0)=\dfrac{8}{\sqrt{3}}+4$

 Vậy ta có điều cần chứng minh. Dấu "=" xảy ra khi $a=b=\dfrac{1}{2},c=0$ cùng các hoán vị

 

 Bài toán 17. (Sưu tầm) Cho các số thực không âm $x,y,z$ thỏa mãn $x+y+z=32$. Tìm giá trị lớn nhất của

$P=x^3y+y^3z+z^3x$




#635435 VMF's Marathon Bất Đẳng Thức Olympic

Đã gửi bởi hoanglong2k on 25-05-2016 - 14:35 trong Bất đẳng thức và cực trị

Mình biết là vậy nhưng mình nghĩ sau khi đồng bậc thì nên giữ lại điều kiện vì rõ ràng bất đẳng thức sau không đúng với mọi số thực.

Và đây là một cách chứng minh khác

$(a^4+b^4+c^4)+abc(a+b+c)-5(a^2b^2+b^2c^2+c^2a^2)=\sum (ab+bc+ca)(a-b)^2+\sum \frac{1}{2}(2a+2b-c)^2(a-b)^2= \sum 3(a-b)^2+\sum \frac{1}{2}(2a+2b-c)^2(a-b)^2 \geq 0$

P/s: mời bạn hoanglong2k đề xuất bài mới  :D

 Ngẫm lại thì mình nghĩ bài này có thêm điều kiện $ab+bc+ca\geq 0$ nữa mới chuẩn được

 

 Bài toán 7. (VMF) Cho $a,b,c\geq 0$ trong đó không có hai số nào đồng thời bằng 0 và số thực $k$ thoả mãn $3^k\geq 2^{k+1}$. Chứng minh rằng : 

\[\frac{1}{a^k+b^k}+\frac{1}{b^k+c^k}+\frac{1}{c^k+a^k}\geq \frac{5.2^{k-1}}{(a+b+c)^k}\]

 

 \begin{array}{| l | l |} \hline \text{HDTterence2k} & 1\\ \hline \text{hoanglong2k} & 3\\ \hline \text{Gachdptrai12} & 2\\ \hline \text{Nguyenhuyen_AG} & 3\\ \hline \text{fatcat12345} & 1\\ \hline \text{lenhatsinh3} & 1\\ \hline\end{array} 




#635428 VMF's Marathon Bất Đẳng Thức Olympic

Đã gửi bởi hoanglong2k on 25-05-2016 - 14:16 trong Bất đẳng thức và cực trị

 Lời giải bài 6. Đưa bài toán về đồng bậc là

 $4(a^4+b^4+c^4)+11abc(a+b+c)\geq 5(ab+bc+ca)^2$

 $\Leftrightarrow 4(a^4+b^4+c^4)+abc(a+b+c)\geq 5(a^2b^2+b^2c^2+c^2a^2)$

 Với $ab+bc+ca>0$

 Đến đây có thể sử dụng ABC, hoặc bổ đề tổng bình phương của thầy Cẩn, mình xin trình bày một cách khác

 Do bậc của bất đẳng thức chẵn nên ta hoàn toàn có thể chuẩn hóa cho $a+b+c=3$

 Khi đó, tồn tại số thực $t\in [0;1)$ sao cho $q=ab+bc+ca=3-3t^2$, ta viết lại bất đẳng thức cần chứng minh thành: 

 $44(81-36q+2q^2+12r)+3r\geq 5(q^2-6r)\Leftrightarrow t^4+14t^2-3+3r\geq 0$

 Mặt khác, chú ý với phép đặt này thì $abc\geq (1-2t)(1+t)^2$ cho nên $t^4+14t^2-3+3r\geq t^2(1-t)(5-t)\geq 0$

 Vậy ta có điều cần chứng minh. Dấu "=" xảy ra khi $a=b=c=1$

 

Nếu bất đẳng thức này đúng thì bất đẳng thức sau cũng đúng

\[4(a^4+b^4+c^4)+11abc(a+b+c) \geqslant 5(ab+bc+ca)^2. \quad (1)\]

Cho $a=1,b=1$ thì $(1)$ trở thành

\[8+4c^4+11c(c+2) \geqslant 5(2c+1)^2,\]

hay là

\[(2c+3)(2c+1)(c-1)^2 \geqslant 0.\]

Dễ thấy bất đẳng thức này không phải luôn đúng.

 Em nghĩ việc này có chút khuất mắc, ví dụ như khi thay $3=ab+bc+ca$ thì bất đẳng thức trên không còn hiển nhiên đúng với $a,b,c$ thực bất kì nữa

 Giả dụ bất đẳng thức của anh sai khi $a=b=1,c=-1$ thì ta sẽ tìm một bộ số tỉ lệ $\dfrac{x}{1}=\dfrac{y}{1}=\dfrac{z}{-1}$ và $xy+yz+zx=3$, tuy nhiên lại không tồn tại bộ số nào như vậy trong TH này?

 Vả lại em nghĩ điều kiện bài này phải có $ab+bc+ca>0$

 

Nếu $a=b=1$ thì $c=1$ theo điều kiện rồi mà anh   :D

 Cái đó đồng bậc mà bạn :D




#635378 VMF's Marathon Bất Đẳng Thức Olympic

Đã gửi bởi hoanglong2k on 25-05-2016 - 10:32 trong Bất đẳng thức và cực trị

 

  Bài toán 4. Với $a,b,c\geq 0$ và $ab+bc+ac=2+abc$. Chứng minh:

\[\dfrac{ab(2-c)}{a^2+abc+b^2}+\dfrac{bc(2-a)}{b^2+abc+c^2}+\dfrac{ac(2-b)}{a^2+abc+c^2} \leq 1\]

 Lời giải bài 4. Ta có $\sum \dfrac{ab(2-c)}{a^2+abc+b^2}\leq 1\Leftrightarrow \sum \dfrac{(a+b)^2}{a^2+abc+b^2}\leq 4$

 Áp dụng bất đẳng thức Cauchy-Schwarz ta có $\sum \dfrac{(a+b)^2}{a^2+abc+b^2}\leq \sum \left(\dfrac{a^2}{a^2+\dfrac{abc}{2}}+\dfrac{b^2}{b^2+\dfrac{abc}{2}}\right)=\sum \left(\dfrac{2a}{2a+bc}+\dfrac{2b}{2b+ac}\right)=4\sum \dfrac{a}{2a+bc}$

 Nên ta chỉ cần chứng minh $\dfrac{a}{2a+bc}+\dfrac{b}{2b+ca}+\dfrac{c}{2c+ab}\leq 4$

 Tương đương $\sum a(2b+ca)(2c+ab)\leq (2a+bc)(2b+ca)(2c+ab)$

 $\Leftrightarrow \sum (4abc+a^3bc+2a^2c^2+2a^2b^2)\leq 8abc+4\sum a^2b^2+2abc\sum a^2+a^2b^2c^2$

 Hay là $abc(a^2+b^2+c^2+abc-4)\geq 0$

 Nếu $abc=0$ thì bất đẳng thức hiển nhiên đúng

 Nếu $abc\neq 0$, theo nguyên lý Dirichlet giả sử $(a-1)(b-1)\geq 0\Rightarrow ab\geq a+b-1\Leftrightarrow abc\geq ac+bc-c\Leftrightarrow ab+c\geq 2\Leftrightarrow abc+c^2\geq 2c$

 Áp dụng bất đẳng thức AM-GM ta có $a^2+b^2+c^2+abc\geq a^2+b^2+2c\geq 2(ab+c)\geq 4$

 Từ đó ta có điều cần chứng minh. Dấu "=" xảy ra khi $a=b=c=1$ hoặc $a=b=\sqrt{2},c=0$ cùng các hoán vị

 

 Bài toán 5. (Phạm Kim Hùng) Cho các số thực $a,b,c$ dương. Tìm hằng số $k$ tốt nhất sao cho bất đẳng thức sau luôn đúng

\[\dfrac{a^3+b^3+c^3}{(a+b)(b+c)(c+a)}+\dfrac{k(ab+bc+ca)}{(a+b+c)^2}\geq \dfrac{3}{8}+\dfrac{k}{3}\]

 




#635333 VMF's Marathon Bất Đẳng Thức Olympic

Đã gửi bởi hoanglong2k on 24-05-2016 - 23:40 trong Bất đẳng thức và cực trị

P/s. Tóm lại giá trị lớn nhất và nhỏ nhất cụ thể của bài 2 là bao nhiêu vậy nhỉ ?

 Em đã chỉnh lại rồi ạ :) Mà tiện thể anh đề xuất bài toán số 4 luôn đi ạ ?

 

 Anh Huyện nhanh tay thật ..

 Cách 2 cho bài toán 3:

 Từ giả thiết ta thu được: $\sum { \frac { 1 }{ a^{ 4 }+1 }  } =1.$

 Áp dụng bất đẳng thức Cauchy Schwarz:

 $1=\sum { \frac { \frac { 1 }{ { a }^{ 4 } }  }{ \frac { 1 }{ { a }^{ 4 } } +1 } \ge \frac { { (\sum { \frac { 1 }{ { a }^{ 2 } } ) }  }^{ 2 } }{ \sum { \frac { 1 }{ { a }^{ 4 } } +3 }  }  } $

 $\Leftrightarrow \sum \frac{1}{a^4}+3\ge (\sum \frac{1}{a^2})^2\Leftrightarrow 3\ge 2\sum \frac{1}{a^2b^2}$

 Sử dụng AM-GM ta có

 $ 9\ge 6\sum { \frac { 1 }{ { a }^{ 2 }{ b }^{ 2 } } \ge 2{ \frac { { (a+b+c) }^{ 2 } }{ (abc)^{ 2 } }  } }$

 $\Leftrightarrow { 9(abc) }^{ 2 }\ge 2{ (a+b+c) }^{ 2 }\Leftrightarrow  3abc\ge \sqrt { 2 } (a+b+c)\ \ (1)$

 Mặt khác, áp dụng bất đẳng thức AM-GM và kết hợp (1): $\frac { abc(a+b+c) }{ ab+bc+ac } \ge \frac { \sqrt { 2 } { (a+b+c) }^{ 2 } }{ 3(ab+bc+ac) } \ge \sqrt { 2 }$

 Dấu bằng xảy ra tại $a=b=c=\sqrt [ 4 ]{ 2 }$

 

 Mình thấy cách này khác gì cách anh Huyện đâu :)

 

\begin{array}{| l | l |} \hline \text{HDTterence2k} & 1\\ \hline \text{hoanglong2k} & 1\\ \hline \text{Gachdptrai12} & 1\\ \hline \text{Nguyenhuyen_AG} & 2\\ \hline\end{array} 




#635303 Marathon số học Olympic

Đã gửi bởi hoanglong2k on 24-05-2016 - 22:14 trong Số học

Để xử lí việc này. ta có thể gải sử a,b thuộc [0,1). sau đó giả sử a và b khác 0, xét 2 trường hợp : trường hợp 1: là a và b hữu tỉ suy ra mâu thuẫn, trường hợp 2 : xét a hoặc b vô tỷ ( đây là việc khó nhất trong bài) cũng suy ra mâu thuẫn.

  Vấn đề chính là nằm ở chỗ $a$ và $b$ cùng là số vô tỉ đó bạn, chỗ này mình không làm được 




#635293 VMF's Marathon Bất Đẳng Thức Olympic

Đã gửi bởi hoanglong2k on 24-05-2016 - 21:53 trong Bất đẳng thức và cực trị

  Bài toán 2. (Phạm Kim Hùng) Cho $a,b,c\geq 0$ và $a+b+c=1$. Tìm giá trị lớn nhất và nhỏ nhất của \[P=\dfrac { a-b }{ \sqrt { a+b }  } +\dfrac { b-c }{ \sqrt { b+c }  } +\dfrac { c-a }{ \sqrt { a+c }  } \]

 Lời giải bài 2. Không mất tính tổng quát, giả sử $a=\min \{a,b,c\}$, đặt $f(a,b,c)=\dfrac{a-b}{\sqrt{a+b}}+\dfrac{b-c}{\sqrt{b+c}}+\dfrac{c-a}{\sqrt{a+c}}$

 Khi đó $f(0,a+b,c)=\dfrac{-a-b}{\sqrt{a+b}}+a+b-c+\sqrt{c}$ và $f(0,b,a+c)=-\sqrt{b}+b-a-c+\dfrac{a+c}{\sqrt{a+c}}$

 - Nếu $b\geq c\geq a$ ta có

$\begin{align*}f(a,b,c)-f(0,a+b,c)&=\dfrac{2a}{\sqrt{a+b}}+\dfrac{b-c}{\sqrt{b+c}}-(b-c)-a+\dfrac{c-a}{\sqrt{c+a}}-\sqrt{c}\\ &=(b-c).\dfrac{1-\sqrt{b+c}}{\sqrt{b+c}}+\dfrac{a}{\sqrt{a+b}}-a+\dfrac{a}{\sqrt{a+b}}-\dfrac{a}{\sqrt{a+c}}+\dfrac{c}{\sqrt{c+a}}-\sqrt{c}\\ &=\dfrac{a(b-c)}{b+c+\sqrt{b+c}}+\dfrac{a(1-\sqrt{a+b})}{\sqrt{a+b}}+\dfrac{a(\sqrt{a+c}-\sqrt{a+b})}{\sqrt{(a+b)(a+c)}}+\sqrt{c}\left (\dfrac{\sqrt{c}}{\sqrt{c+a}}-1\right)\\ &=\dfrac{a(b-c)}{b+c+\sqrt{b+c}}+\dfrac{ac}{a+b+\sqrt{a+b}}+\dfrac{a(c-b)}{\sqrt{(a+b)(a+c)}(\sqrt{a+c}+\sqrt{a+b})}+\dfrac{\sqrt{c}(-a)}{c+a+\sqrt{c(c+a)}}\\ &=aA \end{align*}$

   Trong đó

$A=(b-c)\left[ \dfrac{1}{b+c+\sqrt{b+c}}-\dfrac{1}{\sqrt{(a+b)(a+c)}(\sqrt{a+c}+\sqrt{a+b})}\right ]+\sqrt{c}\left[\dfrac{\sqrt{c}}{a+b+\sqrt{a+b}}-\dfrac{1}{c+a+\sqrt{c(c+a)}}\right ]$

   Mà ta lần lượt có $b-c\geq 0, \sqrt{(a+b)(a+c)}(\sqrt{a+c}+\sqrt{a+b})\leq 2(b+c)\leq b+c+\sqrt{b+c}$

   Và $c\sqrt{c}+a\sqrt{c}+c\sqrt{c+a}\leq c+a+\sqrt{c+a}\leq b+a+\sqrt{a+b}$

   Nên $A\leq 0$ hay $f(a,b,c)\leq f(0,a+b,c)$

   Mặt khác, $f(0,a+b,c)=\dfrac{-a-b}{\sqrt{a+b}}+a+b-c+\sqrt{c}=1-2c+\sqrt{c}-\sqrt{1-c}$

   Đặt $g(c)=1-2c+\sqrt{c}-\sqrt{1-c}$ là hàm số liên tục trên $[0,1]$, có $g'(c)=\dfrac{1}{2\sqrt{c}}+\dfrac{1}{2\sqrt{1-c}}-2$

   Lại có $g'(c)=0\Leftrightarrow c=\dfrac{8\pm \sqrt{46-2\sqrt{17}}}{16}$

   Từ đó dễ dàng kiểm tra được $g(c)\leq g\left(\dfrac{8-\sqrt{46-2\sqrt{17}}}{16}\right)$

   Suy ra $P_\max =\dfrac{\sqrt{46-2\sqrt{17}}}{8}+\dfrac{\sqrt{8-\sqrt{46-2\sqrt{17}}}-\sqrt{8+\sqrt{46-2\sqrt{17}}}}{4}$

   Dấu "=" xảy ra khi $(a,b,c)=\left(0,\dfrac{8+\sqrt{46-2\sqrt{17}}}{16},\dfrac{8-\sqrt{46-2\sqrt{17}}}{16}\right)$ cùng các hoán vị tương ứng

 - Nếu $c\geq b\geq a$, tương tự ta chứng minh được $f(a,b,c)\geq f(0,b,c+a)=2b-1+\sqrt{1-b}-\sqrt{b}=h(b)\geq h\left(\dfrac{8-\sqrt{46-2\sqrt{17}}}{16}\right)$

    Suy ra $P_\min =-\dfrac{\sqrt{46-2\sqrt{17}}}{8}+\dfrac{\sqrt{8+\sqrt{46-2\sqrt{17}}}-\sqrt{8-\sqrt{46-2\sqrt{17}}}}{4}$

    Dấu "=" xảy ra khi $(a,b,c)=\left(0,\dfrac{8-\sqrt{46-2\sqrt{17}}}{16},\dfrac{8+\sqrt{46-2\sqrt{17}}}{16}\right)$ cùng các hoán vị tương ứng

 

Tái bút

 Bài toán 3. Cho các số thực dương $a,b,c$ thỏa mãn $\dfrac{1}{a^4+1}+\dfrac{1}{b^4+1}=\dfrac{c^4}{c^4+1}$. Chứng minh rằng

\[\dfrac{abc(a+b+c)}{ab+bc+ca}\geq \sqrt{2}\]




#635101 VMF's Marathon Bất Đẳng Thức Olympic

Đã gửi bởi hoanglong2k on 23-05-2016 - 22:58 trong Bất đẳng thức và cực trị

 Chào các bạn :D Tình hình là Marathon Số học và Hình học đã mở ra rồi, mình và bạn Gachdptrai12 cũng quyết định lên tiếng cho nó rôm rả

 (Thú thực là mình thấy bên AoPS và có ý đinh lâu rồi nhưng không biết tổ chứ thế nào, nay có Ego mở đầu nên cũng an tâm)

 Thể lệ thì các bạn sẽ xem thêm ở Topic này, nhớ đọc kĩ và chú ý là do Topic mình lập được đặt ở box THCS nên mình sẽ phạt thẳng tay những bạn không tuân thủ :-| Và mình sẽ rút kinh nghiệm không giống như "Tiếp sức BĐT" đâu nhé!

 

 Cuối cùng, mình biết BĐT là phần mà có lẽ nhiều bạn thảo luận nên sẽ phát triển ơn được đâu :-(

 Bài toán hiện tại (37).(AoPS) Cho $a,b,c$ là các số thực dương thỏa  $a^{2}+b^{2}+c^{2}=3$ chứng minh rằng

 $\frac{a^{2}+3b^{2}}{a+3b}+\frac{b^{2}+3c^{2}}{b+3c}+\frac{c^{2}+3a^{2}}{c+3a}\geq 3$

 

 Bài toán 1. (Võ Quốc Bá Cẩn) Cho $a,b,c\geq 0$. Chứng minh rằng \[\sqrt{\dfrac{a(b+c)}{b^2+c^2}}+\sqrt{\dfrac{b(c+a)}{c^2+a^2}}+\sqrt{\dfrac{c(a+b)}{a^2+b^2}}\geq 2\]

 

 

 

Điểm




#634998 Marathon số học Olympic

Đã gửi bởi hoanglong2k on 23-05-2016 - 17:44 trong Số học

 Lời giải bài 3. Do $2^na+b$ là số chính phương với mọi $n$ nên $2^{n+2}a+4b$ và $2^{n+2}a+b$ cũng là số chính phương.

  Đặt $x^2=2^{n+2}a+4b$ và $y^2=2^{n+2}a+b$

  Nếu $a<0$, chọn $n$ đủ lớn để $2^na+b<0$ sẽ vô lí, nên $a\geq 0$, xét khi $a\neq 0$

  Với $b>0$, chọn $n$ sao cho $2^{n}> \dfrac{9b^2-10b+1}{16a}$, khi đó ta sẽ có 

  $4.2^{n+2}a>9b^2-10b+1\Leftrightarrow 4x^2>(3b+1)^2\Leftrightarrow 2x>3b+1\Leftrightarrow 2^{n+2}a+b>2^{n+2}a+4b-2x+1$

  $\Leftrightarrow y^2>(x-1)^2$

  Mặt khác thì $b>0$ nên $(x-y)(x+y)=3b>0$ nên $x>y$, hay $x^2>y^2>(x-1)^2$, vô lí

  Với $b<0$ thì $-3b-1>0$, chọn $n$ sao cho $2^{n}> \dfrac{9b^2-10b+1}{16a}$, khi đó ta sẽ có 

  $4.2^{n+2}a>9b^2-10b+1\Leftrightarrow 4x^2>(3b+1)^2\Leftrightarrow 2x>-3b-1\Leftrightarrow 2^{n+2}a+4b+2x+1>2^{n+2}a+b$

  $\Leftrightarrow (x+1)^2>y^2$

  Mặt khác $b<0$ nên $x<y$ hay $x^2<y^2<(x+1)^2$ vô lí

  Do đó $b=0$ hay $2^na$ là số chính phương với mọi $n$

  Cho $n=0$ và $n=1$ ta được $q^2=2a=2p^2$, điều này xảy ra khi và chỉ khi $p=q=0$ tức là $a=0$

 

 Chả biết nên đề xuất bài như thế nào :-s

 Bài 4. (AoPS - Bulgaria 1983) Chứng minh rằng nếu $a,b,c$ là các số thực thỏa mãn $[na]+[nb]=[nc]$ với mọi $n$ nguyên dương thì tồn tại ít nhất một trong ba số $a,b,c$ nguyên




#634277 Cho $d\mid n$. Khi đó $\forall n\in N^*$ t...

Đã gửi bởi hoanglong2k on 20-05-2016 - 14:00 trong Số học

Cho $d\mid n$. Khi đó $\forall n\in N^*$ ta được $\sum_{d}\varphi(d)=n$

 Đặt $A=\{1;2;...;n-1;n\}$.

 Chia tập $A$ thành các tập $A_i$ với $i=\overline{1,n}$ thỏa mãn $A_i=\left \{k\in A\mid (n,k)=i\right \}=\left \{k\in A\mid \left (\dfrac{n}{i},\dfrac{k}{i}\right )=1\right \}$

 Khi đó, ta có $\dfrac{k}{i}\leq \dfrac{n}{i}$ và $ \left (\dfrac{n}{i},\dfrac{k}{i}\right )=1$ nên $|A_i|=\varphi \left (\dfrac{n}{i}\right )$

 Từ đó suy ra $n=|A|=\sum_{i\mid n} |A_i|=\sum_{i\mid n} \varphi \left (\dfrac{n}{i}\right )$, chọn $i=\dfrac{n}{d}$ ta có điều cần chứng minh




#633987 $a+\frac{1}{b},b+\frac{1}{c...

Đã gửi bởi hoanglong2k on 18-05-2016 - 22:21 trong Số học

Tìm tất cả số hữu tỉ a,b,c dương sao cho $a+\frac{1}{b},b+\frac{1}{c},c+\frac{1}{a}$ nguyên dương

 Từ giả thiết ta suy ra $\mathbb{Z^+}\ni \left(a+\dfrac{1}{b}\right)\left(b+\dfrac{1}{c}\right)\left(c+\dfrac{1}{a}\right)=a+\dfrac{1}{b}+b+\dfrac{1}{c}+c+\dfrac{1}{a}+abc+\dfrac{1}{abc}$

 Suy ra $abc+\dfrac{1}{abc}\in \mathbb{Z^+}$, đặt $abc+\dfrac{1}{abc}=t\geq 2$ thì $(abc)^2-tabc+1=0$

 Có $\Delta =t^2-4$ nên phương trình có nghiệm $abc=\dfrac{t\pm \sqrt{t^2-4}}{2}$, mà $a,b,c\in \mathbb{Q^+}$ nên $abc\in \mathbb{Q^+}$, hay $t^2-4$ phải là số chính phương.

 Đặt $t^2-4=k^2$ thì $(t-k)(t+k)=4$, mà $t+k$ và $t-k$ cùng tính chẵn lẻ nên $t-k=t+k=2\Rightarrow t=2\Rightarrow abc=1$

 Mặt khác ta có $a+\dfrac{1}{b},\ b+\dfrac{1}{c},\ c+\dfrac{1}{a}\in \mathbb{Z^+}\Rightarrow \dfrac{ab+b+1}{b},\ \dfrac{bc+c+1}{c},\ \dfrac{ca+a+1}{a}\in \mathbb{Z^+}$

 Mà từ $abc=1$ thì $\dfrac{b}{ab+b+1}+\dfrac{c}{bc+c+1}+\dfrac{a}{ca+a+1}=1$ nên bài toán quy về tìm các số nguyên dương $x,y,z$ sao cho $\dfrac{1}{x}+\dfrac{1}{y}+\dfrac{1}{z}=1$

 

Đoạn này để giải cái phương trình trên, không thích có thể không ấn vào

 Ta thu được các nghiệm $(x,y,z)\in \{(6,3,2);(4,4,2);(3,3,3)\}$ cùng các hoán vị tương ứng.

   - Nếu $(x,y,z)=(6,3,2)$ thì ta có thể giả sử $\dfrac{ab+b+1}{b}=6,\ \dfrac{bc+c+1}{c}=3,\ \dfrac{ca+a+1}{a}=2$ hay $a+\dfrac{1}{b}=5,\ b+\dfrac{1}{c}=3,\ c+\dfrac{1}{a}=1$

     Thay $b=\dfrac{1}{5-a}$ và $\dfrac{1}{c}=\dfrac{a}{a-1}$ ta được $\dfrac{1}{5-a}+\dfrac{a}{a-1}=3\Rightarrow a=3\Rightarrow b=\dfrac{1}{2},\ c=\dfrac{2}{3}$

     Do đó ta có $(a,b,c)=\left(3,\dfrac{1}{2},\dfrac{2}{3}\right)$ cùng các hoán vị tương ứng

   - Nếu $(x,y,z)=(4,4,2)$, tương tự ta giả sử và thu được $a+\dfrac{1}{b}=3,\ b+\dfrac{1}{c}=2,\ c+\dfrac{1}{a}=1$

     Thay $b=\dfrac{1}{3-a}$ và $\dfrac{1}{c}=\dfrac{a}{a-1}$ ta được $\dfrac{1}{3-a}+\dfrac{a}{a-1}=2\Rightarrow a=2\Rightarrow b=1,\ c=\dfrac{1}{2}$

     Do đó ta có $(a,b,c)=\left(2,1,\dfrac{1}{2}\right)$ cùng các hoán vị tương ứng

   - Nếu $(x,y,z)=(3,3,3)$, thì $a+\dfrac{1}{b}=b+\dfrac{1}{c}=c+\dfrac{1}{a}=2$, đến đây áp dụng AM-GM ta được $(a,b,c)=(1,1,1)$

 Vậy $(a,b,c)\in \left \{\left(3,\dfrac{1}{2},\dfrac{2}{3}\right);\left(2,1,\dfrac{1}{2}\right);(1,1,1)\right \}$ cùng các hoán vị tương ứng